效用

如何計算此特定效用函式的馬歇爾需求

  • October 3, 2016

$$ u(x) = x_1^\alpha \cdot (x_2+x_3)^{1-\alpha}, \text{ with } \alpha \in(0,1) $$ 我試圖設置拉格朗日,結果是 $ \lambda $ 沒有解決方案,除非 $ p_1 = p_2 $ .

$$ L = x_1^\alpha \cdot (x_2+x_3)^{1-\alpha} -\lambda(x_1 p_1 + x_2 p_2 + x_3 p_3 -1) $$ $$ \frac{\partial L}{\partial x_2} = (1-\alpha)x_1^\alpha(x_2+x_3)^{-\alpha} -\lambda p_2 = 0 $$ $$ \frac{\partial L}{\partial x_3} = (1-\alpha)x_1^\alpha(x_2+x_3)^{-\alpha} -\lambda p_3 = 0 $$ 我注意到這個功能類似於 CD 實用功能。所以我想知道我是否可以結合 $ x_{3} $ 和 $ x_{2} $ 作為一種商品,然後應用 CS 效用函式。但是這種方法對我來說似乎不可靠。

任何人都可以幫忙嗎?

這意味著水平面 $ u $ 不要接觸表面 $ x_1p_1+x_2p_2+x_3p_3=1 $ 除非 $ p_2=p_3 $ . 但由於 $ x_i\ge0 $ , 由此可見的允許域 $ x_i $ 是緊湊的,最大值仍然在某個地方實現。即,在允許域的某個邊界點 $ - $ 為了 $ x_2=0 $ 或者 $ x_3=0 $ .

$ x_2 $ 和 $ x_3 $ 是完美的替代品。的確,我總能帶走一單位 $ x_2 $ 並給出一個額外的單位 $ x_3 $ 效用將完全保持不變。這意味著消費者只會購買 $ x_2 $ 或者 $ x_3 $ 最便宜。換句話說,如果 $ p_2\leq p_3 $ 那麼設置沒有一般性損失 $ x_3=0 $ . 那麼問題就變成了

$$ \max_{x_1,x_2}x_1^a x_2^{1-a} $$ 受制於 $$ p_1 x_1+p_2 x_2=M $$ 這是您已經知道如何求解的標準 Cobb-Douglas 函式。

這實際上是一個非常巧妙的問題,因為它迫使我們思考消費者在最大化效用時所做的機制,而不是盲目地設置優化問題。

引用自:https://economics.stackexchange.com/questions/13602